LSAT and Law School Admissions Forum

Get expert LSAT preparation and law school admissions advice from PowerScore Test Preparation.

User avatar
 KelseyWoods
PowerScore Staff
  • PowerScore Staff
  • Posts: 1079
  • Joined: Jun 26, 2013
|
#71167
Hi Hope!

You're absolutely right that in Weaken/Family 3 questions, we're looking for new information in the correct answer that attacks our argument (and in fact the correct answer here, (E), provides new information). The key, however, is that the new information must be relevant to the argument in order to attack it. If the new information addresses something that is not present in the argument, then that new information is irrelevant and will not attack the argument.

Answer choice (A) does not help us attack this argument because we do not know what the economic conditions in this city are. (A) just tells us that in OTHER cities, there's a correlation between increasing economic conditions and decreasing crime rate. But we have no information as to whether or not the economic conditions in the specific city in our stimulus are increasing so it is not relevant to our argument. For this to present a true alternate cause, we would have to know, either from the stimulus or the answer choice, whether that alternate cause existed in this city (in other words, that economic conditions in this city are increasing).

Hope this helps!

Best,
Kelsey
 claudiagarin
  • Posts: 18
  • Joined: May 18, 2020
|
#75606
Hi there,

I have read the above response as to why B is incorrect but I am still unsure. Wouldn't B prove that the new enacted law is not the cause for decreased violent crime, as judges have already been imposing harsh sentencing for a few years? Wouldn't it disprove the conclusion and indicate that this is not the cause for decreased violent crime?
 Christen Hammock
PowerScore Staff
  • PowerScore Staff
  • Posts: 61
  • Joined: May 14, 2020
|
#75634
Hey Claudia!

I think there are two reasons why Answer Choice (B) doesn't weaken the conclusion. First, there's no indication that this judicial practice changed at all in the year before! If something that happened for years before continued to happen in the previous year, there's no reason to think that it could result in the 15% drop in violent crime. Second, as Adam explained above, Answer Choice (B) is too vague to tell us anything about its impact on violent crime rates! We have no idea about the types of crimes they sentenced harshly, how often they did so, or whether those "unusually harsh" penalties involved jail time. Imagine a judge who only handles low level, non-violent crimes but imposes high fines. That would be an "unusually harsh" penalty but wouldn't impact the violent crime rate at all!
 TigerPrince
  • Posts: 5
  • Joined: Apr 14, 2020
|
#77830
Hello!

I understand why (E) is correct, because the policy is still in effect in the year the argument is concerned with, and so that could be an alternate cause that produces the effect of a 15% decrease in violent crime.

I initially chose (C), however, because I thought that this answer choice shows that an effect of a decrease in crime occurred even though the cause that the argument states, i.e., the sentencing law, didn't exist at the time. Since this is a weaken question with cause and effect elements, I went with (C).

However, I might be able to see why (C) is incorrect: (1) It concerns last year, which is irrelevant to the argument's focus on this year. (2) The effect referenced in the argument is a 15% drop, not a 5%, so even though there was a drop both last year and this year, the effect of a 15% drop never occurred last year. Therefore, I cannot make the argument that Cause(x) --> Effect.

Is my reasoning alright? Thanks for any help!

Joshua
User avatar
 KelseyWoods
PowerScore Staff
  • PowerScore Staff
  • Posts: 1079
  • Joined: Jun 26, 2013
|
#77896
Hi Joshua!

Careful with your analysis here: answer choice (C) does concern "last year," but so does the argument in the stimulus! ("Last year our city experienced...", "At the beginning of that year...", "Since no other policy changes were made last year..."). Both answer choice (C) and the stimulus are discussing what occurred over the last year. So that is not a good reason to dismiss answer choice (C).

The real problem with answer choice (C) is that it is referring to the overall crime rate, whereas our stimulus argument is specifically about the violent crime rate. The causal argument that we are trying to weaken is that the mandatory sentencing law caused the 15% decrease in the rate of violent crime.

As Adam said in a previous post:
Adam Tyson wrote: Answer C also doesn't hurt the causal claim. If violent crime is down 15 percent, and overall crime is down 5 percent, that could mean any number of things. Maybe nonviolent crime increased? Maybe violent crime has always made up a relatively small percentage of total crime, and so it's reduction had a disproportionately low impact on the total? We don't know what these varying percentages tell us, but they definitely do NOT suggest that there was some other cause, or that the cause was present without the effect, for example. Remember, the policy enacted applied only to violent crimes, so changes in the rate of nonviolent crime, if any, are outside the scope of the causal argument here. Maybe the harsher penalties for violent crimes caused some criminals to switch from carjacking and mugging to identity theft and embezzlement? Maybe, but the new policy could still be the cause of the decrease in the rate of violent crime.
Hope this helps!

Best,
Kelsey
 TigerPrince
  • Posts: 5
  • Joined: Apr 14, 2020
|
#77915
Kelsey,

I see now—thank you! I should keep a better track of the details in the stimulus. I understand why (C) wouldn't weaken the argument.

Joshua
User avatar
 daultonhaynes
  • Posts: 2
  • Joined: Feb 22, 2021
|
#84373
Hello, I am still confused on why E is correct, even after reading a lot of these comments, and need a little more help in thinking this through.

When I initially read that more officers would be hired I associated that with the idea that because there were more officers on the streets, more offenders would be caught, therefore increasing the percentage. Am I wrong in my assumption that catching more criminals would lead in an uptick of reported violent crimes? Are we to assume that regardless of the # of police that all violent crimes will be reported and accounted for?

Additionally, is E correct because the notion that more officers were on the streets previous years meant more offenders were caught and the amount of possible violent crime offenders on the street had decreased? I feel like this assumes that at least or more than 15% more violent crime offenders had to have been reduced from the pool of possible violent crime offenders to counteract 1. more officers catching more violent crime offenders therefore there being more crime 2. new violent crime committers not in the previous pools.

This really seems to be a convoluted conclusion as opposed to the notion that the economy of the city increased therefore decreasing crime rates (I chose A). However, I can very much be overthinking this and could be the one making this convoluted. Any help straightening out my line of thinking would be very much appreciated
 Jeremy Press
PowerScore Staff
  • PowerScore Staff
  • Posts: 1000
  • Joined: Jun 12, 2017
|
#84395
Hi daulton,

You're overthinking (a common problem for students starting out on Weaken and Strengthen questions!).

Let's start with answer choice A. Yes, that answer says that studies have shown correlations between improving economies and decreased crime rates. But is there any evidence (either within that answer choice or within the stimulus) that the economy has improved in the Police Commissioner's city? I can't find any (and I'm betting you can't either ;) ). And we're not allowed to add information beyond the answer/stimulus to "help" an answer choice on Weaken/Strengthen questions. Yes, the answer itself can bring in new information. But this answer didn't bring in any information about whether this city's economy is improving (or has improved). So get rid of it!

Now, since this is a Weaken question with a Cause/Effect conclusion (the conclusion being that the new mandatory sentencing law caused the decrease in the violent crime rate), a correct answer could be that something is happening in this city that might (key word, might!) be an alternate cause of the decrease in violent crime rate. Answer choice E talks about something happening in this city: 100 new police officers were hired this year. Might that be an alternate cause of a decrease in the violent crime rate? Sure! New police officers on the job might prevent violent crimes, decreasing the violent crime rate.

Now, what you're probably feeling unsatisfied with is your very natural intuition that having new police officers doesn't HAVE TO result in a decrease in the violent crime. And that's true. It doesn't have to. But it could, couldn't it? Yes, and since it could, it makes a dent in the conclusion (weakens it).

The correct answer to a Weaken question doesn't have to destroy the conclusion, just dent it.

I hope this helps!
User avatar
 daultonhaynes
  • Posts: 2
  • Joined: Feb 22, 2021
|
#84505
Jeremy Press wrote:Hi daulton,

You're overthinking (a common problem for students starting out on Weaken and Strengthen questions!).

Let's start with answer choice A. Yes, that answer says that studies have shown correlations between improving economies and decreased crime rates. But is there any evidence (either within that answer choice or within the stimulus) that the economy has improved in the Police Commissioner's city? I can't find any (and I'm betting you can't either ;) ). And we're not allowed to add information beyond the answer/stimulus to "help" an answer choice on Weaken/Strengthen questions. Yes, the answer itself can bring in new information. But this answer didn't bring in any information about whether this city's economy is improving (or has improved). So get rid of it!

Now, since this is a Weaken question with a Cause/Effect conclusion (the conclusion being that the new mandatory sentencing law caused the decrease in the violent crime rate), a correct answer could be that something is happening in this city that might (key word, might!) be an alternate cause of the decrease in violent crime rate. Answer choice E talks about something happening in this city: 100 new police officers were hired this year. Might that be an alternate cause of a decrease in the violent crime rate? Sure! New police officers on the job might prevent violent crimes, decreasing the violent crime rate.

Now, what you're probably feeling unsatisfied with is your very natural intuition that having new police officers doesn't HAVE TO result in a decrease in the violent crime. And that's true. It doesn't have to. But it could, couldn't it? Yes, and since it could, it makes a dent in the conclusion (weakens it).

The correct answer to a Weaken question doesn't have to destroy the conclusion, just dent it.

I hope this helps!
Hi Jeremy, yeah, it kind of does. I figured I was overthinking- honestly this is the first question I've seen on the LSAT that I truly just disagree with/cannot get myself to really rationalize. What you're saying absolutely makes sense, but even knowing the right answer it's still hard to convince myself it is! Oh well, hopefully this is a one time issue and obviously one other students are having as well.

Thank you very much for taking the time to reply! I really appreciate it :-D
User avatar
 MusaMuneer9898
  • Posts: 6
  • Joined: Jul 14, 2021
|
#89011
I have read all of the responses regarding why answer choice (B) is incorrect. However, I believe I can still argue that it is a possible alternative cause just the way (E) is, despite the weak language ("Some crimes", "Many years").

My reasoning goes that since judges, for many years, had already imposed unusually harsh penalties for some crimes, it is possible that these crimes included violent crimes. Also, it could be further possible, for example, that the "many years" refers to the three years leading up to the enactment of the mandatory sentencing law. Therefore, it is possible that these harsh penalties that were imposed for the past 3 years, for example, before the mandatory sentencing policy was enacted, resulted in a 15% decrease in the violent crime rate that specific year because it deterred criminals by imposing harsh penalties for the prior three years.

I understand my support is contingent on a lot of possibilities, but (E) is as well.

Thank you :)

Get the most out of your LSAT Prep Plus subscription.

Analyze and track your performance with our Testing and Analytics Package.